https://www.homydezign.com/science/30155814/
1. Which set illustrates P U M where set P={a, b, c, d, e} and set M={a, e, i, o, u}? A. (a, b, c, d, e, i, o, u} C. (a, b, c, d, e B. (a, e, i, o, u} D. {a, e}